Determine all primitive functions

Click For Summary
The discussion focuses on finding all primitive functions, or antiderivatives, for the function 2x(x^2 + 3)^4. A participant initially attempted to expand the expression but received incorrect results, indicating a misunderstanding of the problem. The correct approach involves using substitution, specifically letting z = x^2 + 3, which simplifies the integration process. The highest degree term in the final answer should be x^10, and it is crucial to include the constant of integration. The thread emphasizes the importance of substitution and the chain rule in reverse for solving such integrals.
beyondlight
Messages
64
Reaction score
0

Homework Statement



Determine all primitive functions for the function:

2x(x^2+3)^4

2. The attempt at a solution

When i expanded i got the primitive to be:

2(x^9/9)+3x^8+18x^6+54x^4+81x^2But this was wrong. I am not sure I have understood the question. Help?
 
Physics news on Phys.org
beyondlight said:

Homework Statement



Determine all primitive functions for the function:

2x(x^2+3)^4

2. The attempt at a solution

When i expanded i got the primitive to be:

2(x^9/9)+3x^8+18x^6+54x^4+81x^2But this was wrong. I am not sure I have understood the question. Help?
I have moved this thread, as it is not a precalculus problem.

Apparently the problem asks you to find all antiderivatives of 2x(x2 + 3)4. In other words, carry out this integration: ##\int 2x(x^2 + 3)^4 dx##. The answer should have a highest degree term of x10. Don't forget the constant of integration.
Instead of expanding the binomial in parentheses, think about a simple substitution that you can do.
 
One possible substitution is 2x(z)^4

But I am not sure how to proceed from here...
 
beyondlight said:
One possible substitution is 2x(z)^4
If z = x^2 + 3, what is dz?
beyondlight said:
But I am not sure how to proceed from here...
A very simple substitution will work, and you're on the right track,
When you use substitution to evaluate an integral, you're using the chain rule in reverse.
 
Question: A clock's minute hand has length 4 and its hour hand has length 3. What is the distance between the tips at the moment when it is increasing most rapidly?(Putnam Exam Question) Answer: Making assumption that both the hands moves at constant angular velocities, the answer is ## \sqrt{7} .## But don't you think this assumption is somewhat doubtful and wrong?

Similar threads

  • · Replies 4 ·
Replies
4
Views
4K
  • · Replies 1 ·
Replies
1
Views
2K
  • · Replies 2 ·
Replies
2
Views
2K
  • · Replies 14 ·
Replies
14
Views
2K
  • · Replies 4 ·
Replies
4
Views
4K
Replies
1
Views
2K
  • · Replies 6 ·
Replies
6
Views
2K
  • · Replies 10 ·
Replies
10
Views
2K
Replies
5
Views
2K
  • · Replies 1 ·
Replies
1
Views
2K